LSAT and Law School Admissions Forum

Get expert LSAT preparation and law school admissions advice from PowerScore Test Preparation.

 ylikate
  • Posts: 30
  • Joined: Aug 27, 2013
|
#10893
Have a few questions regarding this comparative passage set

Q3: Can someone explain to me why A is the correct AC. Passage B never discuss Mars Reconnaissance Orbiter



Thank you in advance!
 Steve Stein
PowerScore Staff
  • PowerScore Staff
  • Posts: 1153
  • Joined: Apr 11, 2011
|
#10894
Hi ylikate,

For question #3 from that passage set, the answer comes down to the tricky term "consistency."

To be "consistent" with a passage simply means that there are no inconsistencies--note that the question doesn't say supported by passage B, so the topic need not have been discussed.

I hope that's helpful! Please let me know whether this is clear--thanks!

~Steve
 Nadia0702
  • Posts: 53
  • Joined: Sep 19, 2013
|
#12352
Hi Steve,
Can you please explain why answer choice E is wrong?

Thanks,
Nadia
 Jacques Lamothe
PowerScore Staff
  • PowerScore Staff
  • Posts: 50
  • Joined: Sep 24, 2013
|
#12354
Hey Nadia,

While passage A mentions that 'Festooning' is a pattern that can appear on rocks, it never says that it is a pattern that appears on the surface of rocks. Lines 16-18 suggest that Festooning is a pattern that occurs "within layers of rock." So even though passage B does not contain information that is inconsistent with answer choice (E), passage A does not support (E).

I hope that helps!

Jacques
 Nadia0702
  • Posts: 53
  • Joined: Sep 19, 2013
|
#12365
Yep, I see it now. Thanks Jacques!
 Blueballoon5%
  • Posts: 156
  • Joined: Jul 13, 2015
|
#27606
Question:

3.) The correct answer is answer choice A. This answer choice refers to "the evidence under discussion." This evidence is supposed to be "consistent with passage B" (according to the stimulus). However, passage B has another evidence (in the second paragraph), not only the evidence from the Mars Reconnaissance Orbiter. How can the correct answer choice only focus on one evidence under discussion and ignore the second evidence?


Thanks!!
 Shannon Parker
PowerScore Staff
  • PowerScore Staff
  • Posts: 147
  • Joined: Jun 08, 2016
|
#27618
hi there,

Question 3 is asking you which of the following statements is supported by passage A, and is consistent with passage B? The key words in the question are "supported" and "consistent." This means that the answer choice must be affirmatively supported by the first passage, meaning the first passage states it, and that it does not contradict the second passage.

In question 3 the correct answer choice "A" states that " the evidence under discussion was uncovered by there Mars Reconnaissance Orbiter." This is stated in the first sentence of passage A. Passage B includes other evidence but nowhere in the passage does it talk about how that evidence was collected, and therefore the notion that it was collected by the Mars Reconnaissance Orbiter is consistent.


I hope this helps.

~Shannon
 Blueballoon5%
  • Posts: 156
  • Joined: Jul 13, 2015
|
#27630
Hi Shannon!
Thanks so much for answering my questions!!

I have a follow up question to question 3. In passage B, the second paragraph mentions rock samples from mars (this is the second evidence). Even though the passage never mentioned how any of the evidence was obtained, how can we assume that an orbiter could obtain rock samples (I am not sure, but I think an orbiter cannot obtain rock samples)?

Thanks again!
User avatar
 Jonathan Evans
PowerScore Staff
  • PowerScore Staff
  • Posts: 726
  • Joined: Jun 09, 2016
|
#27643
Hi, Blue,

Good question. You do not need to make any assumptions about how rock samples were obtained. In fact, you must not make any such assumptions. As Shannon mentioned, the key words in the question stem are consistent with. Your credited response for question 3 must have explicit support in the first passage but only not contradict or plausibly fit into the second passage. The manner whereby any samples were obtained is not relevant. I hope this helps clear up your question.
 g_lawyered
  • Posts: 211
  • Joined: Sep 14, 2020
|
#88599
Hi P.S.,
Can someone please explain why answer B is incorrect? I picked answer B because I saw it stated in Passage B and thought it was in Paragraph 2 of Passage A when it stated the 2 patterns. Where did I go wrong here? :-?
Thanks in advance

Get the most out of your LSAT Prep Plus subscription.

Analyze and track your performance with our Testing and Analytics Package.